Email
Chat with tutors
logo

Ask Questions, Get Answers

X
 
Answer
Comment
Share
Q)

A random variable $X$ has the following p.d.f\[\]$ \begin{array} {llllllll} \textbf{X:}& 0& 1& 2& 3& 4& 5&6&7 \\ \textbf{P(X):}& 0& k &2k& 2k& 3k& k^{2}&2k^{2}&7k^{2}+k \end{array} $\[\]The value of $k$ is

\[\begin{array}{1 1}(1)\frac{1}{8}&(2)\frac{1}{10}\\(3)0&(4)-1 \;or\; \frac{1}{10}\end{array}\]

1 Answer

Home Ask Tuition Questions
Your payment for is successful.
Continue
...